Which one is the true statement? [closed] The 2019 Stack Overflow Developer Survey Results Are In Unicorn Meta Zoo #1: Why another podcast? Announcing the arrival of Valued Associate #679: Cesar ManaraIs the dog dead or alive?The dark side of the moonWho is the horse thief?Make the statement true!Statements that the deities True and False cannot sayA false and a true statement in the blue-eyed puzzleDetermine which statements are True, and which statements are False5x5 statement tableKnights, Knaves and Normals - the tough oneTrue or Faulse?

Did the UK government pay "millions and millions of dollars" to try to snag Julian Assange?

how can a perfect fourth interval be considered either consonant or dissonant?

Categorical vs continuous feature selection/engineering

Deal with toxic manager when you can't quit

Is it ethical to upload a automatically generated paper to a non peer-reviewed site as part of a larger research?

Why are PDP-7-style microprogrammed instructions out of vogue?

How do you keep chess fun when your opponent constantly beats you?

Could an empire control the whole planet with today's comunication methods?

Can withdrawing asylum be illegal?

Do ℕ, mathbbN, BbbN, symbbN effectively differ, and is there a "canonical" specification of the naturals?

What is the padding with red substance inside of steak packaging?

can infinity be divided by anything?

What options are there, if any, to get information from an enemy's familiar?

One Dimensional Japanese Puzzle

What to do when moving next to a bird sanctuary with a loosely-domesticated cat?

What other Star Trek series did the main TNG cast show up in?

What can I do if neighbor is blocking my solar panels intentionally?

Why don't hard Brexiteers insist on a hard border to prevent illegal immigration after Brexit?

Identify 80s or 90s comics with ripped creatures (not dwarves)

Circular reasoning in L'Hopital's rule

Make it rain characters

number sequence puzzle deep six

Define a list range inside a list

What aspect of planet earth must be changed to prevent the industrial revolution?



Which one is the true statement? [closed]



The 2019 Stack Overflow Developer Survey Results Are In
Unicorn Meta Zoo #1: Why another podcast?
Announcing the arrival of Valued Associate #679: Cesar ManaraIs the dog dead or alive?The dark side of the moonWho is the horse thief?Make the statement true!Statements that the deities True and False cannot sayA false and a true statement in the blue-eyed puzzleDetermine which statements are True, and which statements are False5x5 statement tableKnights, Knaves and Normals - the tough oneTrue or Faulse?










21












$begingroup$


  1. All five statements below are true.

  2. None of the four statements below are true.

  3. Both of the statements above are true.

  4. Exactly one of the three statements above is true.

  5. None of the four statements above are true.

  6. None of the five statements above are true.









share|improve this question











$endgroup$



closed as off-topic by Brandon_J, Alconja, Omega Krypton, Peregrine Rook, Glorfindel Apr 1 at 5:53


This question appears to be off-topic. The users who voted to close gave this specific reason:


  • "This looks like a puzzle you found elsewhere. For content you did not create yourself, proper attribution is required. If you have permission to repost this, please edit to include (at minimum) where it came from, then vote to reopen. Posts which use someone else's content without attribution are generally deleted." – Brandon_J, Alconja, Omega Krypton, Peregrine Rook, Glorfindel
If this question can be reworded to fit the rules in the help center, please edit the question.











  • 1




    $begingroup$
    Ha! Great puzzle! $(+1),colororangebigstar$
    $endgroup$
    – user477343
    Mar 31 at 5:40






  • 3




    $begingroup$
    Is this an original puzzle?
    $endgroup$
    – Dr Xorile
    Mar 31 at 15:04






  • 3




    $begingroup$
    brainly.in/question/8878122
    $endgroup$
    – Paul Evans
    Mar 31 at 22:26






  • 5




    $begingroup$
    @PaulEvans' link suggests that the puzzle has been posted elsewhere before. We require sources to be listed to avoid plagerism. Even if you are the author of the other link, then just let us know that. Or you could cite the other link and state that you came up with it independently. It's a policy of this forum
    $endgroup$
    – Dr Xorile
    Apr 1 at 0:36






  • 3




    $begingroup$
    @giorgircheulishvili Have a look, it's not modified. It's word-for-word identical.
    $endgroup$
    – Paul Evans
    Apr 1 at 12:14
















21












$begingroup$


  1. All five statements below are true.

  2. None of the four statements below are true.

  3. Both of the statements above are true.

  4. Exactly one of the three statements above is true.

  5. None of the four statements above are true.

  6. None of the five statements above are true.









share|improve this question











$endgroup$



closed as off-topic by Brandon_J, Alconja, Omega Krypton, Peregrine Rook, Glorfindel Apr 1 at 5:53


This question appears to be off-topic. The users who voted to close gave this specific reason:


  • "This looks like a puzzle you found elsewhere. For content you did not create yourself, proper attribution is required. If you have permission to repost this, please edit to include (at minimum) where it came from, then vote to reopen. Posts which use someone else's content without attribution are generally deleted." – Brandon_J, Alconja, Omega Krypton, Peregrine Rook, Glorfindel
If this question can be reworded to fit the rules in the help center, please edit the question.











  • 1




    $begingroup$
    Ha! Great puzzle! $(+1),colororangebigstar$
    $endgroup$
    – user477343
    Mar 31 at 5:40






  • 3




    $begingroup$
    Is this an original puzzle?
    $endgroup$
    – Dr Xorile
    Mar 31 at 15:04






  • 3




    $begingroup$
    brainly.in/question/8878122
    $endgroup$
    – Paul Evans
    Mar 31 at 22:26






  • 5




    $begingroup$
    @PaulEvans' link suggests that the puzzle has been posted elsewhere before. We require sources to be listed to avoid plagerism. Even if you are the author of the other link, then just let us know that. Or you could cite the other link and state that you came up with it independently. It's a policy of this forum
    $endgroup$
    – Dr Xorile
    Apr 1 at 0:36






  • 3




    $begingroup$
    @giorgircheulishvili Have a look, it's not modified. It's word-for-word identical.
    $endgroup$
    – Paul Evans
    Apr 1 at 12:14














21












21








21


2



$begingroup$


  1. All five statements below are true.

  2. None of the four statements below are true.

  3. Both of the statements above are true.

  4. Exactly one of the three statements above is true.

  5. None of the four statements above are true.

  6. None of the five statements above are true.









share|improve this question











$endgroup$




  1. All five statements below are true.

  2. None of the four statements below are true.

  3. Both of the statements above are true.

  4. Exactly one of the three statements above is true.

  5. None of the four statements above are true.

  6. None of the five statements above are true.






logical-deduction






share|improve this question















share|improve this question













share|improve this question




share|improve this question








edited Mar 30 at 16:44









Deusovi

62.6k6215269




62.6k6215269










asked Mar 30 at 13:50









giorgi rcheulishviligiorgi rcheulishvili

1216




1216




closed as off-topic by Brandon_J, Alconja, Omega Krypton, Peregrine Rook, Glorfindel Apr 1 at 5:53


This question appears to be off-topic. The users who voted to close gave this specific reason:


  • "This looks like a puzzle you found elsewhere. For content you did not create yourself, proper attribution is required. If you have permission to repost this, please edit to include (at minimum) where it came from, then vote to reopen. Posts which use someone else's content without attribution are generally deleted." – Brandon_J, Alconja, Omega Krypton, Peregrine Rook, Glorfindel
If this question can be reworded to fit the rules in the help center, please edit the question.







closed as off-topic by Brandon_J, Alconja, Omega Krypton, Peregrine Rook, Glorfindel Apr 1 at 5:53


This question appears to be off-topic. The users who voted to close gave this specific reason:


  • "This looks like a puzzle you found elsewhere. For content you did not create yourself, proper attribution is required. If you have permission to repost this, please edit to include (at minimum) where it came from, then vote to reopen. Posts which use someone else's content without attribution are generally deleted." – Brandon_J, Alconja, Omega Krypton, Peregrine Rook, Glorfindel
If this question can be reworded to fit the rules in the help center, please edit the question.







  • 1




    $begingroup$
    Ha! Great puzzle! $(+1),colororangebigstar$
    $endgroup$
    – user477343
    Mar 31 at 5:40






  • 3




    $begingroup$
    Is this an original puzzle?
    $endgroup$
    – Dr Xorile
    Mar 31 at 15:04






  • 3




    $begingroup$
    brainly.in/question/8878122
    $endgroup$
    – Paul Evans
    Mar 31 at 22:26






  • 5




    $begingroup$
    @PaulEvans' link suggests that the puzzle has been posted elsewhere before. We require sources to be listed to avoid plagerism. Even if you are the author of the other link, then just let us know that. Or you could cite the other link and state that you came up with it independently. It's a policy of this forum
    $endgroup$
    – Dr Xorile
    Apr 1 at 0:36






  • 3




    $begingroup$
    @giorgircheulishvili Have a look, it's not modified. It's word-for-word identical.
    $endgroup$
    – Paul Evans
    Apr 1 at 12:14













  • 1




    $begingroup$
    Ha! Great puzzle! $(+1),colororangebigstar$
    $endgroup$
    – user477343
    Mar 31 at 5:40






  • 3




    $begingroup$
    Is this an original puzzle?
    $endgroup$
    – Dr Xorile
    Mar 31 at 15:04






  • 3




    $begingroup$
    brainly.in/question/8878122
    $endgroup$
    – Paul Evans
    Mar 31 at 22:26






  • 5




    $begingroup$
    @PaulEvans' link suggests that the puzzle has been posted elsewhere before. We require sources to be listed to avoid plagerism. Even if you are the author of the other link, then just let us know that. Or you could cite the other link and state that you came up with it independently. It's a policy of this forum
    $endgroup$
    – Dr Xorile
    Apr 1 at 0:36






  • 3




    $begingroup$
    @giorgircheulishvili Have a look, it's not modified. It's word-for-word identical.
    $endgroup$
    – Paul Evans
    Apr 1 at 12:14








1




1




$begingroup$
Ha! Great puzzle! $(+1),colororangebigstar$
$endgroup$
– user477343
Mar 31 at 5:40




$begingroup$
Ha! Great puzzle! $(+1),colororangebigstar$
$endgroup$
– user477343
Mar 31 at 5:40




3




3




$begingroup$
Is this an original puzzle?
$endgroup$
– Dr Xorile
Mar 31 at 15:04




$begingroup$
Is this an original puzzle?
$endgroup$
– Dr Xorile
Mar 31 at 15:04




3




3




$begingroup$
brainly.in/question/8878122
$endgroup$
– Paul Evans
Mar 31 at 22:26




$begingroup$
brainly.in/question/8878122
$endgroup$
– Paul Evans
Mar 31 at 22:26




5




5




$begingroup$
@PaulEvans' link suggests that the puzzle has been posted elsewhere before. We require sources to be listed to avoid plagerism. Even if you are the author of the other link, then just let us know that. Or you could cite the other link and state that you came up with it independently. It's a policy of this forum
$endgroup$
– Dr Xorile
Apr 1 at 0:36




$begingroup$
@PaulEvans' link suggests that the puzzle has been posted elsewhere before. We require sources to be listed to avoid plagerism. Even if you are the author of the other link, then just let us know that. Or you could cite the other link and state that you came up with it independently. It's a policy of this forum
$endgroup$
– Dr Xorile
Apr 1 at 0:36




3




3




$begingroup$
@giorgircheulishvili Have a look, it's not modified. It's word-for-word identical.
$endgroup$
– Paul Evans
Apr 1 at 12:14





$begingroup$
@giorgircheulishvili Have a look, it's not modified. It's word-for-word identical.
$endgroup$
– Paul Evans
Apr 1 at 12:14











8 Answers
8






active

oldest

votes


















12












$begingroup$

This is the line of thought I followed:



Statement #3




is impossible because of #1 and #2 contradicting each other (let's consider only the last three statements, for simplicity). So, #3 must be false.




As a consequence,




#1 must be false.




If #4 were true, then #2 must be true (by exclusion), but this would imply that #4 itself is false. Then,




#4 is false.




If #5 were true,




then #2 must be false. So far, this holds.
If #5 were false, then #2, by exclusion, must be true. But this implies that #3 is true too, which is a contradiction, as seen above.
Then #5 is true, and #2 is false.




Accordingly,




#6 is false because it being true would imply that #5 is false.




In conclusion,




there is only one true statement, as said in the title, and is #5.







share|improve this answer











$endgroup$




















    8












    $begingroup$

    True statement is




    5th statement




    Reason




    1 is false(only 5 is true)

    2 is false(5 is true)

    3 is false(both above are false)

    4 is false(all are false)

    6 is false(5is true)







    share|improve this answer











    $endgroup$








    • 1




      $begingroup$
      (A space after >! is not required. However, you cannot follow a spoiler line immediately with a non-spoiler line; it will break the formatting. A blank line after the spoilered line(s) suffices.)
      $endgroup$
      – Rubio
      Mar 30 at 21:07


















    7












    $begingroup$

    Another nice way to approach this puzzle is by constructing chains of implications. We know there's only one true statement, so if one statement implies another one, then it's false.




    • Firstly, $3Rightarrow1Rightarrow6Rightarrow5$, so $3$ and $1$ and $6$ are false.




    • Since $3$ and $1$ are not true, $4Leftrightarrow2$, so they're both false.




    • The only option left is $5$, so this is the answer.







    share|improve this answer









    $endgroup$












    • $begingroup$
      Except it's also possible to uniquely determine each statement's truth value without assuming the title question implies any fact.
      $endgroup$
      – aschepler
      Mar 31 at 18:40


















    4












    $begingroup$

    The correct one is




    5




    Explanation:




    1 is not possible, as only one is true.

    2 is not possible, as it makes 4 true.

    3 is not possible for similar reasons.

    4 is not true as it makes one of 1, 2 or 3 true as well.

    6 is self-contradictory.







    share|improve this answer









    $endgroup$




















      2












      $begingroup$

      Same answer as everyone else, slightly different reasoning




      1 must be false (if true then 6 would be true and contradict it).

      => 3 is false.

      As a result, if 2 were true then 4 would also be true, contradicting "which one statement", so 2 is false.

      => 4 is false

      Trivially 5 is true, 6 is false.







      share|improve this answer









      $endgroup$




















        1












        $begingroup$

        My Answer




        Statement 5 is the true statement.




        Explanation




        If Statement 1 is true then Statement 6 is true; however, statements 1 and 6 cannot both be true as they are mutually contradictory; therefore, Statement 1 is false.

        If Statement 1 is false then Statement 3 is also false.

        If 2 is true then 4 would be false; However, if statement 4 is false then statement 2 is negated. It is logically impossible for statement 2 to be true.

        If 1, 2, and 3 are false then 4 is also false.

        If 1, 2, 3, and 4 are false then 5 is true.

        Finally, if 5 is true then 6 is false.

        Hence, statement 5 is the only true statement.

        edit: you can also eliminate statements 1 and 3 immediately because they imply that more than one statement is true while the questions states that there is only one true statement.







        share|improve this answer











        $endgroup$




















          1












          $begingroup$

          I have used substitution to determine the only true statement.



          Indeed, I started by writing the logic equivalents of each statement.




          $$1 leftarrow 2 land 3 land 4 land 5 land 6$$
          $$2 leftarrow lnot 3 land lnot 4 land lnot 5 land lnot 6$$
          $$3 leftarrow 1 land 2$$
          $$4 leftarrow (1 land lnot 2 land lnot 3) lor ( lnot 1 land 2 land lnot 3) lor ( lnot 1 land lnot 2 land 3)$$
          $$5 leftarrow lnot 1 land lnot 2 land lnot 3 land lnot 4$$
          $$6 leftarrow lnot 1 land lnot 2 land lnot 3 land lnot 4 land lnot 5$$




          From this, a simple replacement in $6$ gives us




          $$6 leftarrow 5 land lnot 5$$




          Which really is just,




          $$6 leftarrow F$$




          From there, you simply substitute the result in the other equations




          $$1 leftarrow 2 land 3 land 4 land 5 land F $$
          $$2 leftarrow lnot 3 land lnot 4 land lnot 5 land lnot F $$




          Which gives us




          $$ 1 leftarrow F $$




          Again, substitution...




          $$3 leftarrow F land 2$$
          $$4 leftarrow (F land lnot 2 land lnot 3) lor ( lnot F land 2 land lnot 3) lor ( lnot F land lnot 2 land 3)$$
          $$5 leftarrow lnot F land lnot 2 land lnot 3 land lnot 4 $$




          Simplifying to




          $$1 leftarrow F $$
          $$2 leftarrow lnot 4 land lnot 5 $$
          $$3 leftarrow F $$
          $$4 leftarrow (F) lor (2 land T) lor (F) $$
          $$5 leftarrow T land lnot 2 land T land lnot 4 $$
          $$6 leftarrow F $$




          At this point, we can simply rewrite as:




          $$1 leftarrow F $$
          $$2 leftarrow lnot 4 land lnot 5 $$
          $$3 leftarrow F $$
          $$4 leftarrow 2 $$
          $$5 leftarrow lnot 2 $$
          $$6 leftarrow F $$




          This gives us the satisfaction that, in fact,




          $$ 2 leftarrow lnot 2 land lnot lnot 2 $$




          Which is a contradiction, thence




          $$ 2 leftarrow F $$




          Giving us the solution




          $$5 leftarrow T $$







          share|improve this answer









          $endgroup$








          • 1




            $begingroup$
            Welcome to Puzzling SE!
            $endgroup$
            – SteveV
            Mar 31 at 23:55










          • $begingroup$
            Very formal! A superb answer!
            $endgroup$
            – user477343
            Apr 2 at 1:27


















          0












          $begingroup$

          Alright, here's my try. I think I have a fairly straightforward explanation.



          1. All five statements below are true.
          2. None of the four statements below are true.
          3. Both of the statements above are true.
          4. Exactly one of the three statements above is true.
          5. None of the four statements above are true.
          6. None of the five statements above are true.


          We can instantly eliminate




          Statements 1, 3, and 4.




          Why?




          Well, they all say that there is another true answer. The design of the question precludes this from being true - "Which one is the true statement?" (emphasis mine).




          This leaves




          Statements 2, 5, and 6. We need a way to make two of them false. Statement 6 cannot be the true statement - it would make statement 5 true, which would make statement 6 false. Statement 2 and 5 can be true, if the other is false. Ignoring previously eliminated statements, statement 2 says statement 5 is false. Statement 5 says statement 2 is false. However, if statement 2 were true, statement 3 is also true.




          Thus, as a final answer,




          Statement 5 would work.







          share|improve this answer









          $endgroup$



















            8 Answers
            8






            active

            oldest

            votes








            8 Answers
            8






            active

            oldest

            votes









            active

            oldest

            votes






            active

            oldest

            votes









            12












            $begingroup$

            This is the line of thought I followed:



            Statement #3




            is impossible because of #1 and #2 contradicting each other (let's consider only the last three statements, for simplicity). So, #3 must be false.




            As a consequence,




            #1 must be false.




            If #4 were true, then #2 must be true (by exclusion), but this would imply that #4 itself is false. Then,




            #4 is false.




            If #5 were true,




            then #2 must be false. So far, this holds.
            If #5 were false, then #2, by exclusion, must be true. But this implies that #3 is true too, which is a contradiction, as seen above.
            Then #5 is true, and #2 is false.




            Accordingly,




            #6 is false because it being true would imply that #5 is false.




            In conclusion,




            there is only one true statement, as said in the title, and is #5.







            share|improve this answer











            $endgroup$

















              12












              $begingroup$

              This is the line of thought I followed:



              Statement #3




              is impossible because of #1 and #2 contradicting each other (let's consider only the last three statements, for simplicity). So, #3 must be false.




              As a consequence,




              #1 must be false.




              If #4 were true, then #2 must be true (by exclusion), but this would imply that #4 itself is false. Then,




              #4 is false.




              If #5 were true,




              then #2 must be false. So far, this holds.
              If #5 were false, then #2, by exclusion, must be true. But this implies that #3 is true too, which is a contradiction, as seen above.
              Then #5 is true, and #2 is false.




              Accordingly,




              #6 is false because it being true would imply that #5 is false.




              In conclusion,




              there is only one true statement, as said in the title, and is #5.







              share|improve this answer











              $endgroup$















                12












                12








                12





                $begingroup$

                This is the line of thought I followed:



                Statement #3




                is impossible because of #1 and #2 contradicting each other (let's consider only the last three statements, for simplicity). So, #3 must be false.




                As a consequence,




                #1 must be false.




                If #4 were true, then #2 must be true (by exclusion), but this would imply that #4 itself is false. Then,




                #4 is false.




                If #5 were true,




                then #2 must be false. So far, this holds.
                If #5 were false, then #2, by exclusion, must be true. But this implies that #3 is true too, which is a contradiction, as seen above.
                Then #5 is true, and #2 is false.




                Accordingly,




                #6 is false because it being true would imply that #5 is false.




                In conclusion,




                there is only one true statement, as said in the title, and is #5.







                share|improve this answer











                $endgroup$



                This is the line of thought I followed:



                Statement #3




                is impossible because of #1 and #2 contradicting each other (let's consider only the last three statements, for simplicity). So, #3 must be false.




                As a consequence,




                #1 must be false.




                If #4 were true, then #2 must be true (by exclusion), but this would imply that #4 itself is false. Then,




                #4 is false.




                If #5 were true,




                then #2 must be false. So far, this holds.
                If #5 were false, then #2, by exclusion, must be true. But this implies that #3 is true too, which is a contradiction, as seen above.
                Then #5 is true, and #2 is false.




                Accordingly,




                #6 is false because it being true would imply that #5 is false.




                In conclusion,




                there is only one true statement, as said in the title, and is #5.








                share|improve this answer














                share|improve this answer



                share|improve this answer








                edited Mar 30 at 14:45

























                answered Mar 30 at 14:40









                dr01dr01

                646926




                646926





















                    8












                    $begingroup$

                    True statement is




                    5th statement




                    Reason




                    1 is false(only 5 is true)

                    2 is false(5 is true)

                    3 is false(both above are false)

                    4 is false(all are false)

                    6 is false(5is true)







                    share|improve this answer











                    $endgroup$








                    • 1




                      $begingroup$
                      (A space after >! is not required. However, you cannot follow a spoiler line immediately with a non-spoiler line; it will break the formatting. A blank line after the spoilered line(s) suffices.)
                      $endgroup$
                      – Rubio
                      Mar 30 at 21:07















                    8












                    $begingroup$

                    True statement is




                    5th statement




                    Reason




                    1 is false(only 5 is true)

                    2 is false(5 is true)

                    3 is false(both above are false)

                    4 is false(all are false)

                    6 is false(5is true)







                    share|improve this answer











                    $endgroup$








                    • 1




                      $begingroup$
                      (A space after >! is not required. However, you cannot follow a spoiler line immediately with a non-spoiler line; it will break the formatting. A blank line after the spoilered line(s) suffices.)
                      $endgroup$
                      – Rubio
                      Mar 30 at 21:07













                    8












                    8








                    8





                    $begingroup$

                    True statement is




                    5th statement




                    Reason




                    1 is false(only 5 is true)

                    2 is false(5 is true)

                    3 is false(both above are false)

                    4 is false(all are false)

                    6 is false(5is true)







                    share|improve this answer











                    $endgroup$



                    True statement is




                    5th statement




                    Reason




                    1 is false(only 5 is true)

                    2 is false(5 is true)

                    3 is false(both above are false)

                    4 is false(all are false)

                    6 is false(5is true)








                    share|improve this answer














                    share|improve this answer



                    share|improve this answer








                    edited Mar 31 at 1:44

























                    answered Mar 30 at 14:26









                    TojrahTojrah

                    2815




                    2815







                    • 1




                      $begingroup$
                      (A space after >! is not required. However, you cannot follow a spoiler line immediately with a non-spoiler line; it will break the formatting. A blank line after the spoilered line(s) suffices.)
                      $endgroup$
                      – Rubio
                      Mar 30 at 21:07












                    • 1




                      $begingroup$
                      (A space after >! is not required. However, you cannot follow a spoiler line immediately with a non-spoiler line; it will break the formatting. A blank line after the spoilered line(s) suffices.)
                      $endgroup$
                      – Rubio
                      Mar 30 at 21:07







                    1




                    1




                    $begingroup$
                    (A space after >! is not required. However, you cannot follow a spoiler line immediately with a non-spoiler line; it will break the formatting. A blank line after the spoilered line(s) suffices.)
                    $endgroup$
                    – Rubio
                    Mar 30 at 21:07




                    $begingroup$
                    (A space after >! is not required. However, you cannot follow a spoiler line immediately with a non-spoiler line; it will break the formatting. A blank line after the spoilered line(s) suffices.)
                    $endgroup$
                    – Rubio
                    Mar 30 at 21:07











                    7












                    $begingroup$

                    Another nice way to approach this puzzle is by constructing chains of implications. We know there's only one true statement, so if one statement implies another one, then it's false.




                    • Firstly, $3Rightarrow1Rightarrow6Rightarrow5$, so $3$ and $1$ and $6$ are false.




                    • Since $3$ and $1$ are not true, $4Leftrightarrow2$, so they're both false.




                    • The only option left is $5$, so this is the answer.







                    share|improve this answer









                    $endgroup$












                    • $begingroup$
                      Except it's also possible to uniquely determine each statement's truth value without assuming the title question implies any fact.
                      $endgroup$
                      – aschepler
                      Mar 31 at 18:40















                    7












                    $begingroup$

                    Another nice way to approach this puzzle is by constructing chains of implications. We know there's only one true statement, so if one statement implies another one, then it's false.




                    • Firstly, $3Rightarrow1Rightarrow6Rightarrow5$, so $3$ and $1$ and $6$ are false.




                    • Since $3$ and $1$ are not true, $4Leftrightarrow2$, so they're both false.




                    • The only option left is $5$, so this is the answer.







                    share|improve this answer









                    $endgroup$












                    • $begingroup$
                      Except it's also possible to uniquely determine each statement's truth value without assuming the title question implies any fact.
                      $endgroup$
                      – aschepler
                      Mar 31 at 18:40













                    7












                    7








                    7





                    $begingroup$

                    Another nice way to approach this puzzle is by constructing chains of implications. We know there's only one true statement, so if one statement implies another one, then it's false.




                    • Firstly, $3Rightarrow1Rightarrow6Rightarrow5$, so $3$ and $1$ and $6$ are false.




                    • Since $3$ and $1$ are not true, $4Leftrightarrow2$, so they're both false.




                    • The only option left is $5$, so this is the answer.







                    share|improve this answer









                    $endgroup$



                    Another nice way to approach this puzzle is by constructing chains of implications. We know there's only one true statement, so if one statement implies another one, then it's false.




                    • Firstly, $3Rightarrow1Rightarrow6Rightarrow5$, so $3$ and $1$ and $6$ are false.




                    • Since $3$ and $1$ are not true, $4Leftrightarrow2$, so they're both false.




                    • The only option left is $5$, so this is the answer.








                    share|improve this answer












                    share|improve this answer



                    share|improve this answer










                    answered Mar 30 at 16:54









                    Rand al'ThorRand al'Thor

                    71.1k14236472




                    71.1k14236472











                    • $begingroup$
                      Except it's also possible to uniquely determine each statement's truth value without assuming the title question implies any fact.
                      $endgroup$
                      – aschepler
                      Mar 31 at 18:40
















                    • $begingroup$
                      Except it's also possible to uniquely determine each statement's truth value without assuming the title question implies any fact.
                      $endgroup$
                      – aschepler
                      Mar 31 at 18:40















                    $begingroup$
                    Except it's also possible to uniquely determine each statement's truth value without assuming the title question implies any fact.
                    $endgroup$
                    – aschepler
                    Mar 31 at 18:40




                    $begingroup$
                    Except it's also possible to uniquely determine each statement's truth value without assuming the title question implies any fact.
                    $endgroup$
                    – aschepler
                    Mar 31 at 18:40











                    4












                    $begingroup$

                    The correct one is




                    5




                    Explanation:




                    1 is not possible, as only one is true.

                    2 is not possible, as it makes 4 true.

                    3 is not possible for similar reasons.

                    4 is not true as it makes one of 1, 2 or 3 true as well.

                    6 is self-contradictory.







                    share|improve this answer









                    $endgroup$

















                      4












                      $begingroup$

                      The correct one is




                      5




                      Explanation:




                      1 is not possible, as only one is true.

                      2 is not possible, as it makes 4 true.

                      3 is not possible for similar reasons.

                      4 is not true as it makes one of 1, 2 or 3 true as well.

                      6 is self-contradictory.







                      share|improve this answer









                      $endgroup$















                        4












                        4








                        4





                        $begingroup$

                        The correct one is




                        5




                        Explanation:




                        1 is not possible, as only one is true.

                        2 is not possible, as it makes 4 true.

                        3 is not possible for similar reasons.

                        4 is not true as it makes one of 1, 2 or 3 true as well.

                        6 is self-contradictory.







                        share|improve this answer









                        $endgroup$



                        The correct one is




                        5




                        Explanation:




                        1 is not possible, as only one is true.

                        2 is not possible, as it makes 4 true.

                        3 is not possible for similar reasons.

                        4 is not true as it makes one of 1, 2 or 3 true as well.

                        6 is self-contradictory.








                        share|improve this answer












                        share|improve this answer



                        share|improve this answer










                        answered Mar 30 at 14:24









                        Krad CigolKrad Cigol

                        1,056210




                        1,056210





















                            2












                            $begingroup$

                            Same answer as everyone else, slightly different reasoning




                            1 must be false (if true then 6 would be true and contradict it).

                            => 3 is false.

                            As a result, if 2 were true then 4 would also be true, contradicting "which one statement", so 2 is false.

                            => 4 is false

                            Trivially 5 is true, 6 is false.







                            share|improve this answer









                            $endgroup$

















                              2












                              $begingroup$

                              Same answer as everyone else, slightly different reasoning




                              1 must be false (if true then 6 would be true and contradict it).

                              => 3 is false.

                              As a result, if 2 were true then 4 would also be true, contradicting "which one statement", so 2 is false.

                              => 4 is false

                              Trivially 5 is true, 6 is false.







                              share|improve this answer









                              $endgroup$















                                2












                                2








                                2





                                $begingroup$

                                Same answer as everyone else, slightly different reasoning




                                1 must be false (if true then 6 would be true and contradict it).

                                => 3 is false.

                                As a result, if 2 were true then 4 would also be true, contradicting "which one statement", so 2 is false.

                                => 4 is false

                                Trivially 5 is true, 6 is false.







                                share|improve this answer









                                $endgroup$



                                Same answer as everyone else, slightly different reasoning




                                1 must be false (if true then 6 would be true and contradict it).

                                => 3 is false.

                                As a result, if 2 were true then 4 would also be true, contradicting "which one statement", so 2 is false.

                                => 4 is false

                                Trivially 5 is true, 6 is false.








                                share|improve this answer












                                share|improve this answer



                                share|improve this answer










                                answered Mar 31 at 15:51









                                StilezStilez

                                1,224211




                                1,224211





















                                    1












                                    $begingroup$

                                    My Answer




                                    Statement 5 is the true statement.




                                    Explanation




                                    If Statement 1 is true then Statement 6 is true; however, statements 1 and 6 cannot both be true as they are mutually contradictory; therefore, Statement 1 is false.

                                    If Statement 1 is false then Statement 3 is also false.

                                    If 2 is true then 4 would be false; However, if statement 4 is false then statement 2 is negated. It is logically impossible for statement 2 to be true.

                                    If 1, 2, and 3 are false then 4 is also false.

                                    If 1, 2, 3, and 4 are false then 5 is true.

                                    Finally, if 5 is true then 6 is false.

                                    Hence, statement 5 is the only true statement.

                                    edit: you can also eliminate statements 1 and 3 immediately because they imply that more than one statement is true while the questions states that there is only one true statement.







                                    share|improve this answer











                                    $endgroup$

















                                      1












                                      $begingroup$

                                      My Answer




                                      Statement 5 is the true statement.




                                      Explanation




                                      If Statement 1 is true then Statement 6 is true; however, statements 1 and 6 cannot both be true as they are mutually contradictory; therefore, Statement 1 is false.

                                      If Statement 1 is false then Statement 3 is also false.

                                      If 2 is true then 4 would be false; However, if statement 4 is false then statement 2 is negated. It is logically impossible for statement 2 to be true.

                                      If 1, 2, and 3 are false then 4 is also false.

                                      If 1, 2, 3, and 4 are false then 5 is true.

                                      Finally, if 5 is true then 6 is false.

                                      Hence, statement 5 is the only true statement.

                                      edit: you can also eliminate statements 1 and 3 immediately because they imply that more than one statement is true while the questions states that there is only one true statement.







                                      share|improve this answer











                                      $endgroup$















                                        1












                                        1








                                        1





                                        $begingroup$

                                        My Answer




                                        Statement 5 is the true statement.




                                        Explanation




                                        If Statement 1 is true then Statement 6 is true; however, statements 1 and 6 cannot both be true as they are mutually contradictory; therefore, Statement 1 is false.

                                        If Statement 1 is false then Statement 3 is also false.

                                        If 2 is true then 4 would be false; However, if statement 4 is false then statement 2 is negated. It is logically impossible for statement 2 to be true.

                                        If 1, 2, and 3 are false then 4 is also false.

                                        If 1, 2, 3, and 4 are false then 5 is true.

                                        Finally, if 5 is true then 6 is false.

                                        Hence, statement 5 is the only true statement.

                                        edit: you can also eliminate statements 1 and 3 immediately because they imply that more than one statement is true while the questions states that there is only one true statement.







                                        share|improve this answer











                                        $endgroup$



                                        My Answer




                                        Statement 5 is the true statement.




                                        Explanation




                                        If Statement 1 is true then Statement 6 is true; however, statements 1 and 6 cannot both be true as they are mutually contradictory; therefore, Statement 1 is false.

                                        If Statement 1 is false then Statement 3 is also false.

                                        If 2 is true then 4 would be false; However, if statement 4 is false then statement 2 is negated. It is logically impossible for statement 2 to be true.

                                        If 1, 2, and 3 are false then 4 is also false.

                                        If 1, 2, 3, and 4 are false then 5 is true.

                                        Finally, if 5 is true then 6 is false.

                                        Hence, statement 5 is the only true statement.

                                        edit: you can also eliminate statements 1 and 3 immediately because they imply that more than one statement is true while the questions states that there is only one true statement.








                                        share|improve this answer














                                        share|improve this answer



                                        share|improve this answer








                                        edited Mar 31 at 19:39

























                                        answered Mar 31 at 14:56









                                        KRAKRA

                                        113




                                        113





















                                            1












                                            $begingroup$

                                            I have used substitution to determine the only true statement.



                                            Indeed, I started by writing the logic equivalents of each statement.




                                            $$1 leftarrow 2 land 3 land 4 land 5 land 6$$
                                            $$2 leftarrow lnot 3 land lnot 4 land lnot 5 land lnot 6$$
                                            $$3 leftarrow 1 land 2$$
                                            $$4 leftarrow (1 land lnot 2 land lnot 3) lor ( lnot 1 land 2 land lnot 3) lor ( lnot 1 land lnot 2 land 3)$$
                                            $$5 leftarrow lnot 1 land lnot 2 land lnot 3 land lnot 4$$
                                            $$6 leftarrow lnot 1 land lnot 2 land lnot 3 land lnot 4 land lnot 5$$




                                            From this, a simple replacement in $6$ gives us




                                            $$6 leftarrow 5 land lnot 5$$




                                            Which really is just,




                                            $$6 leftarrow F$$




                                            From there, you simply substitute the result in the other equations




                                            $$1 leftarrow 2 land 3 land 4 land 5 land F $$
                                            $$2 leftarrow lnot 3 land lnot 4 land lnot 5 land lnot F $$




                                            Which gives us




                                            $$ 1 leftarrow F $$




                                            Again, substitution...




                                            $$3 leftarrow F land 2$$
                                            $$4 leftarrow (F land lnot 2 land lnot 3) lor ( lnot F land 2 land lnot 3) lor ( lnot F land lnot 2 land 3)$$
                                            $$5 leftarrow lnot F land lnot 2 land lnot 3 land lnot 4 $$




                                            Simplifying to




                                            $$1 leftarrow F $$
                                            $$2 leftarrow lnot 4 land lnot 5 $$
                                            $$3 leftarrow F $$
                                            $$4 leftarrow (F) lor (2 land T) lor (F) $$
                                            $$5 leftarrow T land lnot 2 land T land lnot 4 $$
                                            $$6 leftarrow F $$




                                            At this point, we can simply rewrite as:




                                            $$1 leftarrow F $$
                                            $$2 leftarrow lnot 4 land lnot 5 $$
                                            $$3 leftarrow F $$
                                            $$4 leftarrow 2 $$
                                            $$5 leftarrow lnot 2 $$
                                            $$6 leftarrow F $$




                                            This gives us the satisfaction that, in fact,




                                            $$ 2 leftarrow lnot 2 land lnot lnot 2 $$




                                            Which is a contradiction, thence




                                            $$ 2 leftarrow F $$




                                            Giving us the solution




                                            $$5 leftarrow T $$







                                            share|improve this answer









                                            $endgroup$








                                            • 1




                                              $begingroup$
                                              Welcome to Puzzling SE!
                                              $endgroup$
                                              – SteveV
                                              Mar 31 at 23:55










                                            • $begingroup$
                                              Very formal! A superb answer!
                                              $endgroup$
                                              – user477343
                                              Apr 2 at 1:27















                                            1












                                            $begingroup$

                                            I have used substitution to determine the only true statement.



                                            Indeed, I started by writing the logic equivalents of each statement.




                                            $$1 leftarrow 2 land 3 land 4 land 5 land 6$$
                                            $$2 leftarrow lnot 3 land lnot 4 land lnot 5 land lnot 6$$
                                            $$3 leftarrow 1 land 2$$
                                            $$4 leftarrow (1 land lnot 2 land lnot 3) lor ( lnot 1 land 2 land lnot 3) lor ( lnot 1 land lnot 2 land 3)$$
                                            $$5 leftarrow lnot 1 land lnot 2 land lnot 3 land lnot 4$$
                                            $$6 leftarrow lnot 1 land lnot 2 land lnot 3 land lnot 4 land lnot 5$$




                                            From this, a simple replacement in $6$ gives us




                                            $$6 leftarrow 5 land lnot 5$$




                                            Which really is just,




                                            $$6 leftarrow F$$




                                            From there, you simply substitute the result in the other equations




                                            $$1 leftarrow 2 land 3 land 4 land 5 land F $$
                                            $$2 leftarrow lnot 3 land lnot 4 land lnot 5 land lnot F $$




                                            Which gives us




                                            $$ 1 leftarrow F $$




                                            Again, substitution...




                                            $$3 leftarrow F land 2$$
                                            $$4 leftarrow (F land lnot 2 land lnot 3) lor ( lnot F land 2 land lnot 3) lor ( lnot F land lnot 2 land 3)$$
                                            $$5 leftarrow lnot F land lnot 2 land lnot 3 land lnot 4 $$




                                            Simplifying to




                                            $$1 leftarrow F $$
                                            $$2 leftarrow lnot 4 land lnot 5 $$
                                            $$3 leftarrow F $$
                                            $$4 leftarrow (F) lor (2 land T) lor (F) $$
                                            $$5 leftarrow T land lnot 2 land T land lnot 4 $$
                                            $$6 leftarrow F $$




                                            At this point, we can simply rewrite as:




                                            $$1 leftarrow F $$
                                            $$2 leftarrow lnot 4 land lnot 5 $$
                                            $$3 leftarrow F $$
                                            $$4 leftarrow 2 $$
                                            $$5 leftarrow lnot 2 $$
                                            $$6 leftarrow F $$




                                            This gives us the satisfaction that, in fact,




                                            $$ 2 leftarrow lnot 2 land lnot lnot 2 $$




                                            Which is a contradiction, thence




                                            $$ 2 leftarrow F $$




                                            Giving us the solution




                                            $$5 leftarrow T $$







                                            share|improve this answer









                                            $endgroup$








                                            • 1




                                              $begingroup$
                                              Welcome to Puzzling SE!
                                              $endgroup$
                                              – SteveV
                                              Mar 31 at 23:55










                                            • $begingroup$
                                              Very formal! A superb answer!
                                              $endgroup$
                                              – user477343
                                              Apr 2 at 1:27













                                            1












                                            1








                                            1





                                            $begingroup$

                                            I have used substitution to determine the only true statement.



                                            Indeed, I started by writing the logic equivalents of each statement.




                                            $$1 leftarrow 2 land 3 land 4 land 5 land 6$$
                                            $$2 leftarrow lnot 3 land lnot 4 land lnot 5 land lnot 6$$
                                            $$3 leftarrow 1 land 2$$
                                            $$4 leftarrow (1 land lnot 2 land lnot 3) lor ( lnot 1 land 2 land lnot 3) lor ( lnot 1 land lnot 2 land 3)$$
                                            $$5 leftarrow lnot 1 land lnot 2 land lnot 3 land lnot 4$$
                                            $$6 leftarrow lnot 1 land lnot 2 land lnot 3 land lnot 4 land lnot 5$$




                                            From this, a simple replacement in $6$ gives us




                                            $$6 leftarrow 5 land lnot 5$$




                                            Which really is just,




                                            $$6 leftarrow F$$




                                            From there, you simply substitute the result in the other equations




                                            $$1 leftarrow 2 land 3 land 4 land 5 land F $$
                                            $$2 leftarrow lnot 3 land lnot 4 land lnot 5 land lnot F $$




                                            Which gives us




                                            $$ 1 leftarrow F $$




                                            Again, substitution...




                                            $$3 leftarrow F land 2$$
                                            $$4 leftarrow (F land lnot 2 land lnot 3) lor ( lnot F land 2 land lnot 3) lor ( lnot F land lnot 2 land 3)$$
                                            $$5 leftarrow lnot F land lnot 2 land lnot 3 land lnot 4 $$




                                            Simplifying to




                                            $$1 leftarrow F $$
                                            $$2 leftarrow lnot 4 land lnot 5 $$
                                            $$3 leftarrow F $$
                                            $$4 leftarrow (F) lor (2 land T) lor (F) $$
                                            $$5 leftarrow T land lnot 2 land T land lnot 4 $$
                                            $$6 leftarrow F $$




                                            At this point, we can simply rewrite as:




                                            $$1 leftarrow F $$
                                            $$2 leftarrow lnot 4 land lnot 5 $$
                                            $$3 leftarrow F $$
                                            $$4 leftarrow 2 $$
                                            $$5 leftarrow lnot 2 $$
                                            $$6 leftarrow F $$




                                            This gives us the satisfaction that, in fact,




                                            $$ 2 leftarrow lnot 2 land lnot lnot 2 $$




                                            Which is a contradiction, thence




                                            $$ 2 leftarrow F $$




                                            Giving us the solution




                                            $$5 leftarrow T $$







                                            share|improve this answer









                                            $endgroup$



                                            I have used substitution to determine the only true statement.



                                            Indeed, I started by writing the logic equivalents of each statement.




                                            $$1 leftarrow 2 land 3 land 4 land 5 land 6$$
                                            $$2 leftarrow lnot 3 land lnot 4 land lnot 5 land lnot 6$$
                                            $$3 leftarrow 1 land 2$$
                                            $$4 leftarrow (1 land lnot 2 land lnot 3) lor ( lnot 1 land 2 land lnot 3) lor ( lnot 1 land lnot 2 land 3)$$
                                            $$5 leftarrow lnot 1 land lnot 2 land lnot 3 land lnot 4$$
                                            $$6 leftarrow lnot 1 land lnot 2 land lnot 3 land lnot 4 land lnot 5$$




                                            From this, a simple replacement in $6$ gives us




                                            $$6 leftarrow 5 land lnot 5$$




                                            Which really is just,




                                            $$6 leftarrow F$$




                                            From there, you simply substitute the result in the other equations




                                            $$1 leftarrow 2 land 3 land 4 land 5 land F $$
                                            $$2 leftarrow lnot 3 land lnot 4 land lnot 5 land lnot F $$




                                            Which gives us




                                            $$ 1 leftarrow F $$




                                            Again, substitution...




                                            $$3 leftarrow F land 2$$
                                            $$4 leftarrow (F land lnot 2 land lnot 3) lor ( lnot F land 2 land lnot 3) lor ( lnot F land lnot 2 land 3)$$
                                            $$5 leftarrow lnot F land lnot 2 land lnot 3 land lnot 4 $$




                                            Simplifying to




                                            $$1 leftarrow F $$
                                            $$2 leftarrow lnot 4 land lnot 5 $$
                                            $$3 leftarrow F $$
                                            $$4 leftarrow (F) lor (2 land T) lor (F) $$
                                            $$5 leftarrow T land lnot 2 land T land lnot 4 $$
                                            $$6 leftarrow F $$




                                            At this point, we can simply rewrite as:




                                            $$1 leftarrow F $$
                                            $$2 leftarrow lnot 4 land lnot 5 $$
                                            $$3 leftarrow F $$
                                            $$4 leftarrow 2 $$
                                            $$5 leftarrow lnot 2 $$
                                            $$6 leftarrow F $$




                                            This gives us the satisfaction that, in fact,




                                            $$ 2 leftarrow lnot 2 land lnot lnot 2 $$




                                            Which is a contradiction, thence




                                            $$ 2 leftarrow F $$




                                            Giving us the solution




                                            $$5 leftarrow T $$








                                            share|improve this answer












                                            share|improve this answer



                                            share|improve this answer










                                            answered Mar 31 at 23:36









                                            Theophile DanoTheophile Dano

                                            1112




                                            1112







                                            • 1




                                              $begingroup$
                                              Welcome to Puzzling SE!
                                              $endgroup$
                                              – SteveV
                                              Mar 31 at 23:55










                                            • $begingroup$
                                              Very formal! A superb answer!
                                              $endgroup$
                                              – user477343
                                              Apr 2 at 1:27












                                            • 1




                                              $begingroup$
                                              Welcome to Puzzling SE!
                                              $endgroup$
                                              – SteveV
                                              Mar 31 at 23:55










                                            • $begingroup$
                                              Very formal! A superb answer!
                                              $endgroup$
                                              – user477343
                                              Apr 2 at 1:27







                                            1




                                            1




                                            $begingroup$
                                            Welcome to Puzzling SE!
                                            $endgroup$
                                            – SteveV
                                            Mar 31 at 23:55




                                            $begingroup$
                                            Welcome to Puzzling SE!
                                            $endgroup$
                                            – SteveV
                                            Mar 31 at 23:55












                                            $begingroup$
                                            Very formal! A superb answer!
                                            $endgroup$
                                            – user477343
                                            Apr 2 at 1:27




                                            $begingroup$
                                            Very formal! A superb answer!
                                            $endgroup$
                                            – user477343
                                            Apr 2 at 1:27











                                            0












                                            $begingroup$

                                            Alright, here's my try. I think I have a fairly straightforward explanation.



                                            1. All five statements below are true.
                                            2. None of the four statements below are true.
                                            3. Both of the statements above are true.
                                            4. Exactly one of the three statements above is true.
                                            5. None of the four statements above are true.
                                            6. None of the five statements above are true.


                                            We can instantly eliminate




                                            Statements 1, 3, and 4.




                                            Why?




                                            Well, they all say that there is another true answer. The design of the question precludes this from being true - "Which one is the true statement?" (emphasis mine).




                                            This leaves




                                            Statements 2, 5, and 6. We need a way to make two of them false. Statement 6 cannot be the true statement - it would make statement 5 true, which would make statement 6 false. Statement 2 and 5 can be true, if the other is false. Ignoring previously eliminated statements, statement 2 says statement 5 is false. Statement 5 says statement 2 is false. However, if statement 2 were true, statement 3 is also true.




                                            Thus, as a final answer,




                                            Statement 5 would work.







                                            share|improve this answer









                                            $endgroup$

















                                              0












                                              $begingroup$

                                              Alright, here's my try. I think I have a fairly straightforward explanation.



                                              1. All five statements below are true.
                                              2. None of the four statements below are true.
                                              3. Both of the statements above are true.
                                              4. Exactly one of the three statements above is true.
                                              5. None of the four statements above are true.
                                              6. None of the five statements above are true.


                                              We can instantly eliminate




                                              Statements 1, 3, and 4.




                                              Why?




                                              Well, they all say that there is another true answer. The design of the question precludes this from being true - "Which one is the true statement?" (emphasis mine).




                                              This leaves




                                              Statements 2, 5, and 6. We need a way to make two of them false. Statement 6 cannot be the true statement - it would make statement 5 true, which would make statement 6 false. Statement 2 and 5 can be true, if the other is false. Ignoring previously eliminated statements, statement 2 says statement 5 is false. Statement 5 says statement 2 is false. However, if statement 2 were true, statement 3 is also true.




                                              Thus, as a final answer,




                                              Statement 5 would work.







                                              share|improve this answer









                                              $endgroup$















                                                0












                                                0








                                                0





                                                $begingroup$

                                                Alright, here's my try. I think I have a fairly straightforward explanation.



                                                1. All five statements below are true.
                                                2. None of the four statements below are true.
                                                3. Both of the statements above are true.
                                                4. Exactly one of the three statements above is true.
                                                5. None of the four statements above are true.
                                                6. None of the five statements above are true.


                                                We can instantly eliminate




                                                Statements 1, 3, and 4.




                                                Why?




                                                Well, they all say that there is another true answer. The design of the question precludes this from being true - "Which one is the true statement?" (emphasis mine).




                                                This leaves




                                                Statements 2, 5, and 6. We need a way to make two of them false. Statement 6 cannot be the true statement - it would make statement 5 true, which would make statement 6 false. Statement 2 and 5 can be true, if the other is false. Ignoring previously eliminated statements, statement 2 says statement 5 is false. Statement 5 says statement 2 is false. However, if statement 2 were true, statement 3 is also true.




                                                Thus, as a final answer,




                                                Statement 5 would work.







                                                share|improve this answer









                                                $endgroup$



                                                Alright, here's my try. I think I have a fairly straightforward explanation.



                                                1. All five statements below are true.
                                                2. None of the four statements below are true.
                                                3. Both of the statements above are true.
                                                4. Exactly one of the three statements above is true.
                                                5. None of the four statements above are true.
                                                6. None of the five statements above are true.


                                                We can instantly eliminate




                                                Statements 1, 3, and 4.




                                                Why?




                                                Well, they all say that there is another true answer. The design of the question precludes this from being true - "Which one is the true statement?" (emphasis mine).




                                                This leaves




                                                Statements 2, 5, and 6. We need a way to make two of them false. Statement 6 cannot be the true statement - it would make statement 5 true, which would make statement 6 false. Statement 2 and 5 can be true, if the other is false. Ignoring previously eliminated statements, statement 2 says statement 5 is false. Statement 5 says statement 2 is false. However, if statement 2 were true, statement 3 is also true.




                                                Thus, as a final answer,




                                                Statement 5 would work.








                                                share|improve this answer












                                                share|improve this answer



                                                share|improve this answer










                                                answered Apr 1 at 3:19









                                                Brandon_JBrandon_J

                                                3,702245




                                                3,702245













                                                    Popular posts from this blog

                                                    Adding axes to figuresAdding axes labels to LaTeX figuresLaTeX equivalent of ConTeXt buffersRotate a node but not its content: the case of the ellipse decorationHow to define the default vertical distance between nodes?TikZ scaling graphic and adjust node position and keep font sizeNumerical conditional within tikz keys?adding axes to shapesAlign axes across subfiguresAdding figures with a certain orderLine up nested tikz enviroments or how to get rid of themAdding axes labels to LaTeX figures

                                                    Luettelo Yhdysvaltain laivaston lentotukialuksista Lähteet | Navigointivalikko

                                                    Gary (muusikko) Sisällysluettelo Historia | Rockin' High | Lähteet | Aiheesta muualla | NavigointivalikkoInfobox OKTuomas "Gary" Keskinen Ancaran kitaristiksiProjekti Rockin' High